Which one of the following could be an accurate list of the swimmers of the last five laps, in order from lap 6 throu...

maonuo on September 9, 2020

June2002-S3-Q24

The correct answer does not make sense to me because one of O's laps does not precede immediately before any of J's. Please explain? Thanks

Reply
Create a free account to read and take part in forum discussions.

Already have an account? log in

Emil-Kunkin on November 22 at 04:56PM

Hi, I don't think this is true. The correct answer is c.